Proving if $lim_{nrightarrowinfty}a_n=L $ then $lim_{nrightarrowinfty} frac{a_1+a_2+cdots+a_n}n=L $...












2












$begingroup$



This question already has an answer here:




  • Suppose $lim limits_{n to ∞} a_n=L$. Prove that $limlimits_{n to ∞} frac{a_1+a_2+cdots+a_n}{n}=L$ [duplicate]

    2 answers



  • How to prove that if $lim_{n rightarrow infty}a_n=A$, then $lim_{n rightarrow infty}frac{a_1+…+a_n}{n}=A$ [duplicate]

    3 answers




Good morning, i have a big problem with this proof. I haven't idea about this proof.



Problem:



Suppose $lim_{nrightarrowinfty}a_n =L $ then $lim_{nrightarrowinfty} frac{a_1+a_2+cdots+a_n}n=L $



I've tried this:



$| a_n-L|<varepsilonRightarrow L-varepsilon<a_n<L+varepsilon$ Please help!!










share|cite|improve this question











$endgroup$



marked as duplicate by Zain Patel, Emily, Mark Viola, colormegone, user223391 Jul 5 '16 at 18:42


This question has been asked before and already has an answer. If those answers do not fully address your question, please ask a new question.























    2












    $begingroup$



    This question already has an answer here:




    • Suppose $lim limits_{n to ∞} a_n=L$. Prove that $limlimits_{n to ∞} frac{a_1+a_2+cdots+a_n}{n}=L$ [duplicate]

      2 answers



    • How to prove that if $lim_{n rightarrow infty}a_n=A$, then $lim_{n rightarrow infty}frac{a_1+…+a_n}{n}=A$ [duplicate]

      3 answers




    Good morning, i have a big problem with this proof. I haven't idea about this proof.



    Problem:



    Suppose $lim_{nrightarrowinfty}a_n =L $ then $lim_{nrightarrowinfty} frac{a_1+a_2+cdots+a_n}n=L $



    I've tried this:



    $| a_n-L|<varepsilonRightarrow L-varepsilon<a_n<L+varepsilon$ Please help!!










    share|cite|improve this question











    $endgroup$



    marked as duplicate by Zain Patel, Emily, Mark Viola, colormegone, user223391 Jul 5 '16 at 18:42


    This question has been asked before and already has an answer. If those answers do not fully address your question, please ask a new question.





















      2












      2








      2


      2



      $begingroup$



      This question already has an answer here:




      • Suppose $lim limits_{n to ∞} a_n=L$. Prove that $limlimits_{n to ∞} frac{a_1+a_2+cdots+a_n}{n}=L$ [duplicate]

        2 answers



      • How to prove that if $lim_{n rightarrow infty}a_n=A$, then $lim_{n rightarrow infty}frac{a_1+…+a_n}{n}=A$ [duplicate]

        3 answers




      Good morning, i have a big problem with this proof. I haven't idea about this proof.



      Problem:



      Suppose $lim_{nrightarrowinfty}a_n =L $ then $lim_{nrightarrowinfty} frac{a_1+a_2+cdots+a_n}n=L $



      I've tried this:



      $| a_n-L|<varepsilonRightarrow L-varepsilon<a_n<L+varepsilon$ Please help!!










      share|cite|improve this question











      $endgroup$





      This question already has an answer here:




      • Suppose $lim limits_{n to ∞} a_n=L$. Prove that $limlimits_{n to ∞} frac{a_1+a_2+cdots+a_n}{n}=L$ [duplicate]

        2 answers



      • How to prove that if $lim_{n rightarrow infty}a_n=A$, then $lim_{n rightarrow infty}frac{a_1+…+a_n}{n}=A$ [duplicate]

        3 answers




      Good morning, i have a big problem with this proof. I haven't idea about this proof.



      Problem:



      Suppose $lim_{nrightarrowinfty}a_n =L $ then $lim_{nrightarrowinfty} frac{a_1+a_2+cdots+a_n}n=L $



      I've tried this:



      $| a_n-L|<varepsilonRightarrow L-varepsilon<a_n<L+varepsilon$ Please help!!





      This question already has an answer here:




      • Suppose $lim limits_{n to ∞} a_n=L$. Prove that $limlimits_{n to ∞} frac{a_1+a_2+cdots+a_n}{n}=L$ [duplicate]

        2 answers



      • How to prove that if $lim_{n rightarrow infty}a_n=A$, then $lim_{n rightarrow infty}frac{a_1+…+a_n}{n}=A$ [duplicate]

        3 answers








      real-analysis sequences-and-series limits






      share|cite|improve this question















      share|cite|improve this question













      share|cite|improve this question




      share|cite|improve this question








      edited Jul 5 '16 at 16:09









      Michael Hardy

      1




      1










      asked Jul 5 '16 at 16:00









      Bvss12Bvss12

      1,821619




      1,821619




      marked as duplicate by Zain Patel, Emily, Mark Viola, colormegone, user223391 Jul 5 '16 at 18:42


      This question has been asked before and already has an answer. If those answers do not fully address your question, please ask a new question.









      marked as duplicate by Zain Patel, Emily, Mark Viola, colormegone, user223391 Jul 5 '16 at 18:42


      This question has been asked before and already has an answer. If those answers do not fully address your question, please ask a new question.
























          4 Answers
          4






          active

          oldest

          votes


















          5












          $begingroup$

          Here's a version of the proof that maybe looks a little longer than other versions, but which has the advantage that each step is more or less obvious:



          First, we introduce the notation $$sigma_n=frac{a_1+dots+a_n}{n}.$$



          Lemma If there exists $N$ so that $a_n=0$ for all $n>N$ then $limsigma_n=0$.



          Proof: $$|sigma_n|lefrac{|a_1+dots+a_N|}{n}.$$QED



          Proposition If $lim a_n=0$ then $limsigma_n=0$.



          Proof: Let $epsilon>0$. Choose $N$ so $|a_n|<epsilon$ for all $n>N$. Define $$a_n'=begin{cases}0,&(nle N),
          \a_n,&(n>N).end{cases}$$The lemma shows that $$lim(sigma_n-sigma_n')=0$$(using what should be obvious notation). But $|a_n'|leepsilon$ for every $n$, so $|sigma_n'|<epsilon$ for every $n$, hence $$limsup|sigma_n'|leepsilon.$$Since $sigma_n-sigma_n'to0$ this shows that $limsup|sigma_n|leepsilon$, and since this holds for every $epsilon>0$ it follows that $limsup|sigma_n|=0$, hence $limsigma_n=0$. QED.



          Theorem If $lim a_n=L$ then $lim sigma_n=L$.



          Proof: Let $a_n'=a_n-L$. Then the lemma shows that $limsigma_n'=0$, since $lim a_n'=0$. But $sigma_n=sigma_n'+L$, hence $lim sigma_n=L$. QED






          share|cite|improve this answer









          $endgroup$





















            6












            $begingroup$

            Let $epsilon>0$. Therefore, there exists $N in mathbb{N}$ such that $m> N implies a_m < L+epsilon$. Therefore,



            $$frac{a_1+cdots+a_n}{n}=frac{a_1+cdots +a_N}{n}+frac{a_{N+1}+cdots+a_n}{n}$$
            $$< frac{a_1+cdots +a_N}{n}+frac{(n-(N+1))(L+epsilon)}{n}.$$



            Passing the $limsup$ as $n to infty$, we get



            $$limsuplimits_{n to infty} frac{a_1+cdots+a_n}{n} leq L+epsilon.$$
            Analogously, we can prove
            $$liminflimits_{n to infty} frac{a_1+cdots+a_n}{n} geq L-epsilon.$$
            But this holds for every $epsilon>0.$ Therefore,
            $$limlimits_{n to infty} frac{a_1+cdots+a_n}{n}=L.$$






            share|cite|improve this answer











            $endgroup$





















              0












              $begingroup$

              It is called Cesaro lemma. First, write in detail your hypothesis. Then, split your sum (what is below) into two parts according to your detailed hypothesis.



              Remember that you have to control (ie: make it smaller than $epsilon$) the quantity : $midfrac{1}{n}sum_{i=1}^n a_i - L mid = mid frac{1}{n}sum_{i=1}^n (a_i - L )mid$






              share|cite|improve this answer









              $endgroup$





















                0












                $begingroup$

                We may state Cesàro's lemma in the following form:




                If a real sequence ${a_n}_{ngeq 0}$ is converging to $L$, it is converging to $L$ on average, too.




                The usual $varepsilon-delta$ proof is straightforward but quite lengthy. I will give an argument that is easier to follow, at least in my opinion. A converging sequence is a Cauchy sequence, and and averaged Cauchy sequence is still a Cauchy sequence, hence a converging one. Assuming that
                $$ lim_{nto +infty}frac{a_1+a_2+ldots+a_n}{n}= C neq L $$
                we also have
                $$ lim_{nto +infty}frac{a_{n+1}+a_{n+2}+ldots+a_{2n}}{n}=Cneq L $$
                or:
                $$ lim_{nto infty}frac{1}{n}sum_{k=1}^{n}left(a_{n+k}-Lright)neq 0. $$
                That, however, contradicts the fact that ${a_n}_{ngeq 0}$ is a Cauchy sequence converging to $L$.



                At last, Cesàro's lemma can be seen as a consequence of the Hardy-Littlewood tauberian theorem, too: Karamata's proof is very nice and short.






                share|cite|improve this answer









                $endgroup$









                • 3




                  $begingroup$
                  Is this really much shorter, since you have to prove that an averaged sequence of a Cauchy sequence is again Cauchy?
                  $endgroup$
                  – Ian
                  Jul 5 '16 at 16:28




















                4 Answers
                4






                active

                oldest

                votes








                4 Answers
                4






                active

                oldest

                votes









                active

                oldest

                votes






                active

                oldest

                votes









                5












                $begingroup$

                Here's a version of the proof that maybe looks a little longer than other versions, but which has the advantage that each step is more or less obvious:



                First, we introduce the notation $$sigma_n=frac{a_1+dots+a_n}{n}.$$



                Lemma If there exists $N$ so that $a_n=0$ for all $n>N$ then $limsigma_n=0$.



                Proof: $$|sigma_n|lefrac{|a_1+dots+a_N|}{n}.$$QED



                Proposition If $lim a_n=0$ then $limsigma_n=0$.



                Proof: Let $epsilon>0$. Choose $N$ so $|a_n|<epsilon$ for all $n>N$. Define $$a_n'=begin{cases}0,&(nle N),
                \a_n,&(n>N).end{cases}$$The lemma shows that $$lim(sigma_n-sigma_n')=0$$(using what should be obvious notation). But $|a_n'|leepsilon$ for every $n$, so $|sigma_n'|<epsilon$ for every $n$, hence $$limsup|sigma_n'|leepsilon.$$Since $sigma_n-sigma_n'to0$ this shows that $limsup|sigma_n|leepsilon$, and since this holds for every $epsilon>0$ it follows that $limsup|sigma_n|=0$, hence $limsigma_n=0$. QED.



                Theorem If $lim a_n=L$ then $lim sigma_n=L$.



                Proof: Let $a_n'=a_n-L$. Then the lemma shows that $limsigma_n'=0$, since $lim a_n'=0$. But $sigma_n=sigma_n'+L$, hence $lim sigma_n=L$. QED






                share|cite|improve this answer









                $endgroup$


















                  5












                  $begingroup$

                  Here's a version of the proof that maybe looks a little longer than other versions, but which has the advantage that each step is more or less obvious:



                  First, we introduce the notation $$sigma_n=frac{a_1+dots+a_n}{n}.$$



                  Lemma If there exists $N$ so that $a_n=0$ for all $n>N$ then $limsigma_n=0$.



                  Proof: $$|sigma_n|lefrac{|a_1+dots+a_N|}{n}.$$QED



                  Proposition If $lim a_n=0$ then $limsigma_n=0$.



                  Proof: Let $epsilon>0$. Choose $N$ so $|a_n|<epsilon$ for all $n>N$. Define $$a_n'=begin{cases}0,&(nle N),
                  \a_n,&(n>N).end{cases}$$The lemma shows that $$lim(sigma_n-sigma_n')=0$$(using what should be obvious notation). But $|a_n'|leepsilon$ for every $n$, so $|sigma_n'|<epsilon$ for every $n$, hence $$limsup|sigma_n'|leepsilon.$$Since $sigma_n-sigma_n'to0$ this shows that $limsup|sigma_n|leepsilon$, and since this holds for every $epsilon>0$ it follows that $limsup|sigma_n|=0$, hence $limsigma_n=0$. QED.



                  Theorem If $lim a_n=L$ then $lim sigma_n=L$.



                  Proof: Let $a_n'=a_n-L$. Then the lemma shows that $limsigma_n'=0$, since $lim a_n'=0$. But $sigma_n=sigma_n'+L$, hence $lim sigma_n=L$. QED






                  share|cite|improve this answer









                  $endgroup$
















                    5












                    5








                    5





                    $begingroup$

                    Here's a version of the proof that maybe looks a little longer than other versions, but which has the advantage that each step is more or less obvious:



                    First, we introduce the notation $$sigma_n=frac{a_1+dots+a_n}{n}.$$



                    Lemma If there exists $N$ so that $a_n=0$ for all $n>N$ then $limsigma_n=0$.



                    Proof: $$|sigma_n|lefrac{|a_1+dots+a_N|}{n}.$$QED



                    Proposition If $lim a_n=0$ then $limsigma_n=0$.



                    Proof: Let $epsilon>0$. Choose $N$ so $|a_n|<epsilon$ for all $n>N$. Define $$a_n'=begin{cases}0,&(nle N),
                    \a_n,&(n>N).end{cases}$$The lemma shows that $$lim(sigma_n-sigma_n')=0$$(using what should be obvious notation). But $|a_n'|leepsilon$ for every $n$, so $|sigma_n'|<epsilon$ for every $n$, hence $$limsup|sigma_n'|leepsilon.$$Since $sigma_n-sigma_n'to0$ this shows that $limsup|sigma_n|leepsilon$, and since this holds for every $epsilon>0$ it follows that $limsup|sigma_n|=0$, hence $limsigma_n=0$. QED.



                    Theorem If $lim a_n=L$ then $lim sigma_n=L$.



                    Proof: Let $a_n'=a_n-L$. Then the lemma shows that $limsigma_n'=0$, since $lim a_n'=0$. But $sigma_n=sigma_n'+L$, hence $lim sigma_n=L$. QED






                    share|cite|improve this answer









                    $endgroup$



                    Here's a version of the proof that maybe looks a little longer than other versions, but which has the advantage that each step is more or less obvious:



                    First, we introduce the notation $$sigma_n=frac{a_1+dots+a_n}{n}.$$



                    Lemma If there exists $N$ so that $a_n=0$ for all $n>N$ then $limsigma_n=0$.



                    Proof: $$|sigma_n|lefrac{|a_1+dots+a_N|}{n}.$$QED



                    Proposition If $lim a_n=0$ then $limsigma_n=0$.



                    Proof: Let $epsilon>0$. Choose $N$ so $|a_n|<epsilon$ for all $n>N$. Define $$a_n'=begin{cases}0,&(nle N),
                    \a_n,&(n>N).end{cases}$$The lemma shows that $$lim(sigma_n-sigma_n')=0$$(using what should be obvious notation). But $|a_n'|leepsilon$ for every $n$, so $|sigma_n'|<epsilon$ for every $n$, hence $$limsup|sigma_n'|leepsilon.$$Since $sigma_n-sigma_n'to0$ this shows that $limsup|sigma_n|leepsilon$, and since this holds for every $epsilon>0$ it follows that $limsup|sigma_n|=0$, hence $limsigma_n=0$. QED.



                    Theorem If $lim a_n=L$ then $lim sigma_n=L$.



                    Proof: Let $a_n'=a_n-L$. Then the lemma shows that $limsigma_n'=0$, since $lim a_n'=0$. But $sigma_n=sigma_n'+L$, hence $lim sigma_n=L$. QED







                    share|cite|improve this answer












                    share|cite|improve this answer



                    share|cite|improve this answer










                    answered Jul 5 '16 at 17:11









                    David C. UllrichDavid C. Ullrich

                    61.7k44095




                    61.7k44095























                        6












                        $begingroup$

                        Let $epsilon>0$. Therefore, there exists $N in mathbb{N}$ such that $m> N implies a_m < L+epsilon$. Therefore,



                        $$frac{a_1+cdots+a_n}{n}=frac{a_1+cdots +a_N}{n}+frac{a_{N+1}+cdots+a_n}{n}$$
                        $$< frac{a_1+cdots +a_N}{n}+frac{(n-(N+1))(L+epsilon)}{n}.$$



                        Passing the $limsup$ as $n to infty$, we get



                        $$limsuplimits_{n to infty} frac{a_1+cdots+a_n}{n} leq L+epsilon.$$
                        Analogously, we can prove
                        $$liminflimits_{n to infty} frac{a_1+cdots+a_n}{n} geq L-epsilon.$$
                        But this holds for every $epsilon>0.$ Therefore,
                        $$limlimits_{n to infty} frac{a_1+cdots+a_n}{n}=L.$$






                        share|cite|improve this answer











                        $endgroup$


















                          6












                          $begingroup$

                          Let $epsilon>0$. Therefore, there exists $N in mathbb{N}$ such that $m> N implies a_m < L+epsilon$. Therefore,



                          $$frac{a_1+cdots+a_n}{n}=frac{a_1+cdots +a_N}{n}+frac{a_{N+1}+cdots+a_n}{n}$$
                          $$< frac{a_1+cdots +a_N}{n}+frac{(n-(N+1))(L+epsilon)}{n}.$$



                          Passing the $limsup$ as $n to infty$, we get



                          $$limsuplimits_{n to infty} frac{a_1+cdots+a_n}{n} leq L+epsilon.$$
                          Analogously, we can prove
                          $$liminflimits_{n to infty} frac{a_1+cdots+a_n}{n} geq L-epsilon.$$
                          But this holds for every $epsilon>0.$ Therefore,
                          $$limlimits_{n to infty} frac{a_1+cdots+a_n}{n}=L.$$






                          share|cite|improve this answer











                          $endgroup$
















                            6












                            6








                            6





                            $begingroup$

                            Let $epsilon>0$. Therefore, there exists $N in mathbb{N}$ such that $m> N implies a_m < L+epsilon$. Therefore,



                            $$frac{a_1+cdots+a_n}{n}=frac{a_1+cdots +a_N}{n}+frac{a_{N+1}+cdots+a_n}{n}$$
                            $$< frac{a_1+cdots +a_N}{n}+frac{(n-(N+1))(L+epsilon)}{n}.$$



                            Passing the $limsup$ as $n to infty$, we get



                            $$limsuplimits_{n to infty} frac{a_1+cdots+a_n}{n} leq L+epsilon.$$
                            Analogously, we can prove
                            $$liminflimits_{n to infty} frac{a_1+cdots+a_n}{n} geq L-epsilon.$$
                            But this holds for every $epsilon>0.$ Therefore,
                            $$limlimits_{n to infty} frac{a_1+cdots+a_n}{n}=L.$$






                            share|cite|improve this answer











                            $endgroup$



                            Let $epsilon>0$. Therefore, there exists $N in mathbb{N}$ such that $m> N implies a_m < L+epsilon$. Therefore,



                            $$frac{a_1+cdots+a_n}{n}=frac{a_1+cdots +a_N}{n}+frac{a_{N+1}+cdots+a_n}{n}$$
                            $$< frac{a_1+cdots +a_N}{n}+frac{(n-(N+1))(L+epsilon)}{n}.$$



                            Passing the $limsup$ as $n to infty$, we get



                            $$limsuplimits_{n to infty} frac{a_1+cdots+a_n}{n} leq L+epsilon.$$
                            Analogously, we can prove
                            $$liminflimits_{n to infty} frac{a_1+cdots+a_n}{n} geq L-epsilon.$$
                            But this holds for every $epsilon>0.$ Therefore,
                            $$limlimits_{n to infty} frac{a_1+cdots+a_n}{n}=L.$$







                            share|cite|improve this answer














                            share|cite|improve this answer



                            share|cite|improve this answer








                            edited Aug 23 '16 at 6:41

























                            answered Jul 5 '16 at 16:08









                            Aloizio MacedoAloizio Macedo

                            23.8k24088




                            23.8k24088























                                0












                                $begingroup$

                                It is called Cesaro lemma. First, write in detail your hypothesis. Then, split your sum (what is below) into two parts according to your detailed hypothesis.



                                Remember that you have to control (ie: make it smaller than $epsilon$) the quantity : $midfrac{1}{n}sum_{i=1}^n a_i - L mid = mid frac{1}{n}sum_{i=1}^n (a_i - L )mid$






                                share|cite|improve this answer









                                $endgroup$


















                                  0












                                  $begingroup$

                                  It is called Cesaro lemma. First, write in detail your hypothesis. Then, split your sum (what is below) into two parts according to your detailed hypothesis.



                                  Remember that you have to control (ie: make it smaller than $epsilon$) the quantity : $midfrac{1}{n}sum_{i=1}^n a_i - L mid = mid frac{1}{n}sum_{i=1}^n (a_i - L )mid$






                                  share|cite|improve this answer









                                  $endgroup$
















                                    0












                                    0








                                    0





                                    $begingroup$

                                    It is called Cesaro lemma. First, write in detail your hypothesis. Then, split your sum (what is below) into two parts according to your detailed hypothesis.



                                    Remember that you have to control (ie: make it smaller than $epsilon$) the quantity : $midfrac{1}{n}sum_{i=1}^n a_i - L mid = mid frac{1}{n}sum_{i=1}^n (a_i - L )mid$






                                    share|cite|improve this answer









                                    $endgroup$



                                    It is called Cesaro lemma. First, write in detail your hypothesis. Then, split your sum (what is below) into two parts according to your detailed hypothesis.



                                    Remember that you have to control (ie: make it smaller than $epsilon$) the quantity : $midfrac{1}{n}sum_{i=1}^n a_i - L mid = mid frac{1}{n}sum_{i=1}^n (a_i - L )mid$







                                    share|cite|improve this answer












                                    share|cite|improve this answer



                                    share|cite|improve this answer










                                    answered Jul 5 '16 at 16:08









                                    anonymusanonymus

                                    893413




                                    893413























                                        0












                                        $begingroup$

                                        We may state Cesàro's lemma in the following form:




                                        If a real sequence ${a_n}_{ngeq 0}$ is converging to $L$, it is converging to $L$ on average, too.




                                        The usual $varepsilon-delta$ proof is straightforward but quite lengthy. I will give an argument that is easier to follow, at least in my opinion. A converging sequence is a Cauchy sequence, and and averaged Cauchy sequence is still a Cauchy sequence, hence a converging one. Assuming that
                                        $$ lim_{nto +infty}frac{a_1+a_2+ldots+a_n}{n}= C neq L $$
                                        we also have
                                        $$ lim_{nto +infty}frac{a_{n+1}+a_{n+2}+ldots+a_{2n}}{n}=Cneq L $$
                                        or:
                                        $$ lim_{nto infty}frac{1}{n}sum_{k=1}^{n}left(a_{n+k}-Lright)neq 0. $$
                                        That, however, contradicts the fact that ${a_n}_{ngeq 0}$ is a Cauchy sequence converging to $L$.



                                        At last, Cesàro's lemma can be seen as a consequence of the Hardy-Littlewood tauberian theorem, too: Karamata's proof is very nice and short.






                                        share|cite|improve this answer









                                        $endgroup$









                                        • 3




                                          $begingroup$
                                          Is this really much shorter, since you have to prove that an averaged sequence of a Cauchy sequence is again Cauchy?
                                          $endgroup$
                                          – Ian
                                          Jul 5 '16 at 16:28


















                                        0












                                        $begingroup$

                                        We may state Cesàro's lemma in the following form:




                                        If a real sequence ${a_n}_{ngeq 0}$ is converging to $L$, it is converging to $L$ on average, too.




                                        The usual $varepsilon-delta$ proof is straightforward but quite lengthy. I will give an argument that is easier to follow, at least in my opinion. A converging sequence is a Cauchy sequence, and and averaged Cauchy sequence is still a Cauchy sequence, hence a converging one. Assuming that
                                        $$ lim_{nto +infty}frac{a_1+a_2+ldots+a_n}{n}= C neq L $$
                                        we also have
                                        $$ lim_{nto +infty}frac{a_{n+1}+a_{n+2}+ldots+a_{2n}}{n}=Cneq L $$
                                        or:
                                        $$ lim_{nto infty}frac{1}{n}sum_{k=1}^{n}left(a_{n+k}-Lright)neq 0. $$
                                        That, however, contradicts the fact that ${a_n}_{ngeq 0}$ is a Cauchy sequence converging to $L$.



                                        At last, Cesàro's lemma can be seen as a consequence of the Hardy-Littlewood tauberian theorem, too: Karamata's proof is very nice and short.






                                        share|cite|improve this answer









                                        $endgroup$









                                        • 3




                                          $begingroup$
                                          Is this really much shorter, since you have to prove that an averaged sequence of a Cauchy sequence is again Cauchy?
                                          $endgroup$
                                          – Ian
                                          Jul 5 '16 at 16:28
















                                        0












                                        0








                                        0





                                        $begingroup$

                                        We may state Cesàro's lemma in the following form:




                                        If a real sequence ${a_n}_{ngeq 0}$ is converging to $L$, it is converging to $L$ on average, too.




                                        The usual $varepsilon-delta$ proof is straightforward but quite lengthy. I will give an argument that is easier to follow, at least in my opinion. A converging sequence is a Cauchy sequence, and and averaged Cauchy sequence is still a Cauchy sequence, hence a converging one. Assuming that
                                        $$ lim_{nto +infty}frac{a_1+a_2+ldots+a_n}{n}= C neq L $$
                                        we also have
                                        $$ lim_{nto +infty}frac{a_{n+1}+a_{n+2}+ldots+a_{2n}}{n}=Cneq L $$
                                        or:
                                        $$ lim_{nto infty}frac{1}{n}sum_{k=1}^{n}left(a_{n+k}-Lright)neq 0. $$
                                        That, however, contradicts the fact that ${a_n}_{ngeq 0}$ is a Cauchy sequence converging to $L$.



                                        At last, Cesàro's lemma can be seen as a consequence of the Hardy-Littlewood tauberian theorem, too: Karamata's proof is very nice and short.






                                        share|cite|improve this answer









                                        $endgroup$



                                        We may state Cesàro's lemma in the following form:




                                        If a real sequence ${a_n}_{ngeq 0}$ is converging to $L$, it is converging to $L$ on average, too.




                                        The usual $varepsilon-delta$ proof is straightforward but quite lengthy. I will give an argument that is easier to follow, at least in my opinion. A converging sequence is a Cauchy sequence, and and averaged Cauchy sequence is still a Cauchy sequence, hence a converging one. Assuming that
                                        $$ lim_{nto +infty}frac{a_1+a_2+ldots+a_n}{n}= C neq L $$
                                        we also have
                                        $$ lim_{nto +infty}frac{a_{n+1}+a_{n+2}+ldots+a_{2n}}{n}=Cneq L $$
                                        or:
                                        $$ lim_{nto infty}frac{1}{n}sum_{k=1}^{n}left(a_{n+k}-Lright)neq 0. $$
                                        That, however, contradicts the fact that ${a_n}_{ngeq 0}$ is a Cauchy sequence converging to $L$.



                                        At last, Cesàro's lemma can be seen as a consequence of the Hardy-Littlewood tauberian theorem, too: Karamata's proof is very nice and short.







                                        share|cite|improve this answer












                                        share|cite|improve this answer



                                        share|cite|improve this answer










                                        answered Jul 5 '16 at 16:24









                                        Jack D'AurizioJack D'Aurizio

                                        292k33284673




                                        292k33284673








                                        • 3




                                          $begingroup$
                                          Is this really much shorter, since you have to prove that an averaged sequence of a Cauchy sequence is again Cauchy?
                                          $endgroup$
                                          – Ian
                                          Jul 5 '16 at 16:28
















                                        • 3




                                          $begingroup$
                                          Is this really much shorter, since you have to prove that an averaged sequence of a Cauchy sequence is again Cauchy?
                                          $endgroup$
                                          – Ian
                                          Jul 5 '16 at 16:28










                                        3




                                        3




                                        $begingroup$
                                        Is this really much shorter, since you have to prove that an averaged sequence of a Cauchy sequence is again Cauchy?
                                        $endgroup$
                                        – Ian
                                        Jul 5 '16 at 16:28






                                        $begingroup$
                                        Is this really much shorter, since you have to prove that an averaged sequence of a Cauchy sequence is again Cauchy?
                                        $endgroup$
                                        – Ian
                                        Jul 5 '16 at 16:28





                                        Popular posts from this blog

                                        Biblatex bibliography style without URLs when DOI exists (in Overleaf with Zotero bibliography)

                                        ComboBox Display Member on multiple fields

                                        Is it possible to collect Nectar points via Trainline?